Mathcenter Forum

Mathcenter Forum (https://www.mathcenter.net/forum/index.php)
-   ข้อสอบในโรงเรียน ม.ปลาย (https://www.mathcenter.net/forum/forumdisplay.php?f=21)
-   -   สมาคมคณิตศาสตร์ ม.ปลาย ๒๕๕๘ (https://www.mathcenter.net/forum/showthread.php?t=22970)

กิตติ 22 พฤศจิกายน 2015 21:22

สมาคมคณิตศาสตร์ ม.ปลาย ๒๕๕๘
 
3 ไฟล์และเอกสาร
ขอบคุณไฟล์ภาพจากเพจคณิตม.ปลาย โดยอ.ปิง

กิตติ 22 พฤศจิกายน 2015 21:33

3 ไฟล์และเอกสาร
๑๕หน้า ๓๕ ข้อ

กิตติ 22 พฤศจิกายน 2015 21:34

5 ไฟล์และเอกสาร
ต่อครับ พิมพ์ไว้หน่อยจะได้ส่งรูปได้

กิตติ 22 พฤศจิกายน 2015 21:35

5 ไฟล์และเอกสาร
ล็อตสุดท้ายแล้ว

อัศวินมังกรแดง 22 พฤศจิกายน 2015 21:38

1.ค
2.ค
3.ค
4.ก

~ArT_Ty~ 23 พฤศจิกายน 2015 22:50

ไม่ได้มาแวะเวียนซะนาน 55555

ข้อ 34
ให้ $k \in \mathbb{N} $ สังเกตว่า $\displaystyle{\left\lfloor\,\frac{3}{\sqrt{x}}\right\rfloor=k \geqslant 3}$ ( เพราะว่า $0 < x \leqslant 1$)
เมื่อ $\displaystyle{n \leqslant \frac{3}{\sqrt{x}} < k+1}$ เราจะได้ว่า $\displaystyle{\frac{9}{(k+1)^2} < x \leqslant \frac{9}{k^2}}$

ดังนั้น $$\displaystyle{\int_{\frac{9}{(k+1)^2}}^{\frac{9}{k^2}}\left\lfloor\,\frac{3}{\sqrt{x}}\right\rfloor dx=k\left(\,\frac{9}{k^2}-\frac{9}{(k+1)^2}\right) =\frac{9}{(k+1)^2}+\frac{1}{k(k+1)}}$$

กำหนดลำดับ $a_{n}=\displaystyle{\frac{9}{(n+2)^2}}$ จะได้ว่า $a_{1}=1$ และลำดับนี้ลู่เข้าสู่ 0 เมื่อ $n$ มีค่ามากๆ
เราจะได้ว่า $$B=\int_{0}^{1}\left\lfloor\,\frac{3}{\sqrt{x}}\right\rfloor dx =\sum_{n = 1}^{\infty} \int_{a_{n+1}}^{a_{n}}\frac{3}{\sqrt{x}}dx =9\sum_{n = 1}^{\infty} \frac{1}{(n+3)^2}+9\sum_{n = 1}^{\infty} \frac{1}{(n+2)(n+3)}=9\left(\,\frac{1}{4^2}+\frac{1}{5^2}+\frac{1}{6^2}+\ldots \right)+3 $$

ดังนั้น เราจะได้ว่า $$12A-B=12\left(\,\frac{1}{1^2}+\frac{1}{3^2}+\frac{1}{5^2}+\ldots \right)-9\left(\,\frac{1}{4^2}+\frac{1}{5^2}+\frac{1}{6^2}+\ldots \right)-3=9\left(\,\frac{1}{1^2}+\frac{1}{2^2}+\frac{1}{3^2}\right)-3=\frac{37}{4}$$

ป.ล. ข้ามขั้นตอนจัดรูปตอนท้ายๆมาหน่อยนะครับ :kaka: ถ้าผิดตรงไหนขออภัยด้วย

Thgx0312555 23 พฤศจิกายน 2015 23:22

เฉลยบ้าง 32. Let $a= r\cos \theta, b = r\sin \theta$ $(r= \sqrt{a^2+b^2})$

$M=\bmatrix{r\cos \theta & r\sin \theta \\ -r\sin \theta & r\cos \theta} $
by induction
$M^k=\bmatrix{r^k\cos k\theta & r^k\sin k\theta \\ -r^k\sin k\theta & r^k\cos k\theta} $

$M^9+M^7+M^3+M^5+M=0$

$\Leftrightarrow r^9\cos 9\theta+r^7\cos 7\theta+r^5\cos 5\theta+r^3\cos 3\theta+r\cos\theta=0 \wedge r^9\sin 9\theta+r^7\sin 7\theta+r^5\sin 5\theta+r^3\sin 3\theta+r\sin\theta=0$

$\Leftrightarrow r^9 \mathrm{cis} 9\theta+r^7 \mathrm{cis} 7\theta+r^5 \mathrm{cis} 5\theta+r^3 \mathrm{cis} 3\theta+r \mathrm{cis}\theta=0$

$\Leftrightarrow z^9+z^7+z^5+z^3+z=0$ where $z=r\mathrm{cis}\theta$

มี $z$ 9 คำตอบที่เป็นไปได้

ดังนั้นมีคู่อันดับ $(a,b)$ 9 คู่ และมี $A$ 9 matrices

Thgx0312555 24 พฤศจิกายน 2015 07:50

มีข้อน่าเฉลยหลายข้อจริงๆ ดูข้อนี้ก่อนครับ

35. $\displaystyle \sum_{n \in A} \sin ^2 \left( \dfrac{n \pi}{3003}\right) $
$\displaystyle = \sum_{n \in A} \dfrac{1}{2} - \dfrac{1}{2} \sum_{n \in A} \cos \left( \dfrac{2n \pi}{3003}\right)$

$\displaystyle = \dfrac{1}{2} \cdot (3003)\left(\dfrac{2}{3}\right)\left(\dfrac{6}{7}\right)\left(\dfrac{10}{11}\right)\left(\dfrac{12}{13}\right) - \dfrac{1}{2} \sum_{n \in A} \cos \left( \dfrac{2n \pi}{3003}\right)$

$\displaystyle = 720 - \dfrac{1}{2} \sum_{n \in A} \cos \left( \dfrac{2n \pi}{3003}\right)$

โดย PIE

$\displaystyle \sum_{n \in A} \cos \left( \frac{2n \pi}{3003}\right) = \sum_{1 \le n \le 3003} \cos \left( \frac{2n \pi}{3003}\right) - \sum_{p \in \left\{ 3,7,11,13 \right\}} \sum_{1 \le n \le 3003, p \mid n} \cos \left( \frac{2n \pi}{3003}\right) + \sum_{p_1 < p_2 \in \left\{3,7,11,13 \right\}} \sum_{1 \le n \le 3003, p_1p_2 \mid n} \cos \left( \frac{2n \pi}{3003}\right)$

$\displaystyle - \sum_{p_1 < p_2 < p_3 \in \left\{3,7,11,13 \right\}} \sum_{1 \le n \le 3003, p_1p_2p_3 \mid n} \cos \left( \frac{2n \pi}{3003}\right) + \sum_{1 \le n \le 3003, 3003 \mid n} \cos \left( \frac{2n \pi}{3003}\right)$

$\displaystyle = \sum_{1 \le n \le 3003} \cos \left( \frac{2n \pi}{3003}\right) - \sum_{p \in \left\{ 3,7,11,13 \right\}} \sum_{1 \le n \le 3003/p} \cos \left( \frac{2n \pi}{3003/p}\right) + \sum_{p_1 < p_2 \in \left\{3,7,11,13 \right\}} \sum_{1 \le n \le 3003/p_1p_2} \cos \left( \frac{2n \pi}{3003/p_1p_2}\right)$

$\displaystyle - \sum_{p_1 < p_2 < p_3 \in \left\{3,7,11,13 \right\}} \sum_{1 \le n \le 3003/p_1p_2p_3} \cos \left( \frac{2n \pi}{3003/p_1p_2p_3}\right) + \cos 2n \pi$

$= 0-0+0-0+1 = 1$

จากข้อสังเกตต่อไปนี้ เมื่อ $k>1$, ผลบวกรากของ $x^k-1=0$ คือ $0$

$\displaystyle \sum_{1 \le n \le k} \mathrm{cis} \left( \dfrac{2n\pi}{k} \right) = 0$

จะได้ $\displaystyle \sum_{1 \le n \le k} \cos \left( \dfrac{2n\pi}{k} \right) = 0$

$\displaystyle \sum_{n \in A} \sin ^2 \left( \dfrac{n \pi}{3003}\right) =719.5$

พิมพ์ยากมาก ><

~ArT_Ty~ 24 พฤศจิกายน 2015 09:41

ข้อ 13 ไม่เสียนัยสมมุติให้ $AB=1$ และ $AB$ ถูกแบ่งเป็น 3 ส่วน ยาว $x,y,1-x-y$ ตามลำดับ จะได้ว่า $x+y<1$

เนื่องจากเราสามมารถประกอบเป็นสามเหลี่ยมได้ จะได้ว่า $x+y>1-x-y$ นั่นคือ $\displaystyle{x+y>\frac{1}{2}}$

และยังได้อีกว่า $x<y+1-x-y=1-x$ นั่นคือ $\displaystyle{x<\frac{1}{2}}$ ในทำนองเดียวกันก็จะได้ว่า $\displaystyle{y<\frac{1}{2}}$

เอาเงื่อนไขที่ได้มา plot ในแกนพิกัดฉาก $x,y$ ดูครับ

จะได้ว่าความน่าจะเป็นที่จะได้ไม้ 3 ท่อนมาประกอบเป็น 3 เหลี่ยมได้คือ $$P(E)=\frac{\displaystyle{\frac{1}{2}\times\frac{1}{2}\times\frac{1}{2}}}{\displaystyle{\frac{1}{2}\times 1\times1}}=\frac{1}{4}$$

nooonuii 24 พฤศจิกายน 2015 10:48

ไม่ค่อยมีคนเฉลยเลยครับ รู้สึกว่าที่นี่จะเงียบเหงาไปเยอะเลย

24. $2f(x)= 4|x+2|+6|x-2|+2|x-5|+|x-7|$

ดังนั้นค่าต่ำสุดของ $2f(x)$ เกิดเมื่อ $x=$ ค่ามัธยฐานของข้อมูล $-2,-2,-2,-2,2,2,2,2,2,2,5,5,7$

ซึ่งมีค่าเท่ากับ $2$ จึงได้ $f(x)\geq 13.5$ $\blacksquare$

ฟินิกซ์เหินฟ้า 24 พฤศจิกายน 2015 11:02

ข้อ 28-30
 
28. ตอบ $7$

จัดรูป 2 แบบ $z_1^3=z_2^3$ ได้ $|z_1|=|z_2|$ และ $(z_1-z_2)^2=-z_1z_2$

ได้ $1=|z_1-z_2|^2=|-z_1z_2|=|z_1|^2$ ดังนั้น $|z_1|=|z_2|=1$

ที่โจทย์ให้หาคือ $5|z_1|^2+2(z_1\bar z_2+ z_2 \bar z_1)$

ใช้สมการ $|z_1-z_2|=1$ อีกครั้งได้ว่า $2|z_1|^2-(z_1\bar z_2+ z_2 \bar z_1)=1$

ที่โจทย์ให้หากลายเป็น $9|z_1|^2-2=7$


29. ผมได้ $(m,n)=(10,2)$

ตอบ $12$ มั้ยครับ

ผมทำจนได้ $m =kn ; k\in \mathbf{N} $ แล้วสุ่มไปเรื่อยๆ

30. ตอบ $\dfrac{1}{\sqrt{3}}$ มั้ยครับ

จัดรูปบนล่างได้ $\dfrac{cos(10+2x)-1/2 }{cos(10+2x)+1/2}$

จับ $cos(10+2x)=0$ แล้ว $x= \dfrac{n\pi }{2}- 5$

ฟินิกซ์เหินฟ้า 24 พฤศจิกายน 2015 11:17

ข้อ 10.-11.
 
10. ก. $14$ วิธี ปะครับ
แบ่งกลุ่ม 2กลุ่ม ต่างกัน $\dfrac{8!}{4!4!}=14$ แล้วคนที่บังกันไปสลับกันเอง 1 วิธี

11. ผมลองให้ $z=-w$ สอดคล้องสองสมการพอดีครับ
$z=cis (\dfrac{\pi}{4}), w= cis(\dfrac{5\pi}{4})$
แทนค่าไปตอบ ข. $\sqrt{2}-\sqrt{2}i$

ฟินิกซ์เหินฟ้า 24 พฤศจิกายน 2015 11:30

ข้อ 19,22
 
19. ตอบ $ 1,-2log_3 2-2$

$x=1$ เป็นจริงแน่นอน
take log จัดรูปได้ $(x-1) log 3 = -\dfrac{2x-2}{x+2} log 2 $
ตอนนี้ $x \not= 1$ แล้วแก้สมการ ได้$ x =-2log_3 2 -2$

22. ตอบ $100,031$

แจกของ $ 5$ ชิ้นใส่กล่อง $1$ กล่อง,$2$ กล่อง,... โดยกล่องแรกต้องมีของอย่างน้อย $1$ ชิ้น

ได้ $\displaystyle \binom{4}{4}+\binom{5}{4}+\binom {6}{4}+\binom{7}{4} +\binom{8}{4} = 126 $ วิธี เหลือ $4$ วิธี สำหรับ $6$ กล่อง ไล่ง่ายๆได้ $100031$

23. อันนี้ใช้แคลคูลัสครับ ไม่รู้ผู้รู้จะมีวิธีที่ดีกว่าหรือเปล่านะครับ

สมการแรก $4h+3k=2$
สมการที่สอง,สาม,สี่,ห้า differentiate สมการวงกลมเทียบ $x$ แล้วจัดรูปให้เป็น $\dfrac{dy}{dx}$ แล้วแทนจุดจากสมการสัมผัส
แล้วใช้สมการเส้นสัมผัส

แก้ทุกสมการออกมาได้ $(h,k)=(2,-2),(-4,6)$ แทนค่ากลับไปหา $radius$

ตอบ $(x-2)^2+(y+2)^2=8$ และ $(x+4)^2+(y-6)^2=18$

~ArT_Ty~ 24 พฤศจิกายน 2015 17:10

ลองทำข้อ 23 แบบไม่ใช้แคลคูลัสดูครับ

ให้ $(a,b)$ เป็นพิกัดของจุดศูนย์กลางวงกลม จะได้ว่า $4a+3b-2=0$ นั่นคือ $\displaystyle{b=\frac{2-4a}{3}}$

จะได้ว่า รัศมีของวงกลมคือระยะตั้งฉากจากจุด $(a,b)$ ไปยังเส้นตรงทั้งสองเส้น

นั่นคือ $\displaystyle{r=\frac{\left|\,a+b+4\right| }{\sqrt{2}}}=\frac{\left|\,7a-b+4\right| }{5\sqrt{2}}$

แทนค่า $b$ เข้าไปแล้วแก้สมการออกมาจะได้ว่ามี $(a,b)$ ที่สอดคล้องคือ $(a,b)=(2,-2),(-4,6)$

ทำให้ได้สมการวงกลม 2 สมการคือ $\displaystyle{(x-2)^2+(y+2)^2=8}$ และ $\displaystyle{(x+4)^2+(y-6)^2=18}$

~ArT_Ty~ 24 พฤศจิกายน 2015 17:29

ข้อ 26 เห็นว่าต้องใช้โลปิตาลและก็ดิฟผลหารด้วย (ไม่แน่ใจว่า ม.ปลายสอนมั้ย)

จากโจทย์จะได้ว่า $\displaystyle{\lim_{x \to \infty} x\left(\,\arctan\left(\,\frac{x+1}{x+2}\right)-\arctan\left(\,\frac{x}{x+2}\right) \right) }=\lim_{x \to \infty} \frac{\arctan\left(\,\displaystyle{\frac{x+2}{2x^2+5x+4}}\right) }{\displaystyle{\frac{1}{x}}}$ ซึ่งอยู่ในรูปแบบไม่กำหนด $\displaystyle{\frac{0}{0}}$

ใช้ทฤษฎีบทของโลปิตาลทำการดิฟบนดิฟล่างและจัดรูปจะได้ว่า (ขอข้ามขั้นตอนหน่อยนะครับ จัดรูปไม่ยากมากแต่อาจจะถึกสักหน่อย)

$$\displaystyle{\lim_{x \to \infty} x\left(\,\arctan\left(\,\frac{x+1}{x+2}\right)-\arctan\left(\,\frac{x}{x+2}\right) \right) } = \lim_{x \to \infty} \frac{2x^4+3x^3}{(2x^2+5x+4)^2+(x+2)^2}=\frac{1}{2}$$

ไม่ค่อยแน่ใจเหมือนกัน ผิดขออภัยครับ


เวลาที่แสดงทั้งหมด เป็นเวลาที่ประเทศไทย (GMT +7) ขณะนี้เป็นเวลา 03:16

Powered by vBulletin® Copyright ©2000 - 2024, Jelsoft Enterprises Ltd.
Modified by Jetsada Karnpracha